Discussion Forum

AMC 10 Mock Exam Discussion For 9/12: Question 13

 
 
Picture of John Lensmire
AMC 10 Mock Exam Discussion For 9/12: Question 13
by John Lensmire - Wednesday, September 13, 2023, 1:18 PM
 

AMC 10 Mock Exam Question 13

How many positive integers $n$ satisfy the following condition: $$(130n)^{50} > n^{100} > 2^{200} \text{ ?}$$

$\textbf{(A) } 0\qquad \textbf{(B) } 7\qquad \textbf{(C) } 12\qquad \textbf{(D) } 65\qquad \textbf{(E) } 125$

Submit your answer and solution and explanation below! Solutions will be accepted for 48 hours until 9/14 at 2pm Pacific Time. (There's still time for yesterday's problem too: click here.)

Top solutions for all the Mock Exam questions will be collected and shared as part of a full 25 Question Mock AMC 10 Exam.

Note: The question above is a past AMC problem. Solutions submitted must be written by students. Copied solutions will be disqualified.